Find the lowest terms
fraction that's equal
to 0.20.

Answers

Answer 1
0.20 is equal to 1/5.

Related Questions

Please answer question #2 and please show work

Answers

9514 1404 393

Answer:

  2a. (360x +495)/(8x +11) = 45

  2b. 2((8x+11) +45) = 16x +112

Step-by-step explanation:

a) The area is the product of length and width.

  A = LW

We are given ...

  W = 8x +11

  A = 360x +495

The value of length can be found by solving for L:

  L = A/W

  L = (360x +495)/(8x +11)

The long-division solution to this is shown in the attachment.

  L = 45 . . . . simplified expression for length

__

b) The perimeter of the tank is twice the sum of length and width:

  P = 2(L +W)

Substituting the values for L and W, this is ..

  P = 2(45 +(8x +11)) . . . . expression for length of edge seal

  P = 2(8x +56)

  P = 16x + 112 . . . . . . simplified expression for seal length

−[(−r) + 2p] = −(−r) − 2p

Answers

Answer:

chfdhfjfhbdhfuthdhfuruu

You are going on a long trip and want to calculate your Miles per gallon. When you start your trip your odometer read 95842 miles. At the end of your trip your odometer reads 95975 miles. You started with a full tank and to return to a full tank you put in 7 gallons of gas total. What is your miles per gallon?

Answers

Answer:

19 miles per gallon

Step-by-step explanation:

95975 - 95842 = 133

133 divided by 7 = 19


At 5:00pm the temperature read 10° above zero. At midnight it read 4 ° below zero. Find the change
in temperature.

Answers

The temperature changed a total of 14° over the course of 7 hours with an average change of 2° per hour drop.

A. 2
B. -2
C. 1/2
D. -1/2

Answers

Rate of change is the slope.

The slope is the change in Y over the change in X:

Slope = (5- -3) / (2 - -2)

Slope = 8/4

Slope = 2

Answer: A.2

24-9•2+6÷3 pls help I'm doing homework.​

Answers

Answer:

24-9•2+6÷3 =12 did that help you alot

If you give me the right answer the points are yours

Answers

Answer:

-5+2

-3

Step-by-step explanation:

We start with -5 then add 2

-5+2 = -3

Step-by-step explanation:

Expression = -5 + 2

Sum = -5 + 2 = 3

The radius of a circle is 12cm.A chord of the circle is 16cm long.Calculate the distance of the chord from the centre of the circle.Give the answer to 1 decimal point.

NO LINKS PLEASE!!!

Answers

the distance d of the chord from the center of the circle is:

d^2 = 12^2 - (16/2)^2
d^2 = 144 - 64
d^2 = 80
d = 8.9 cm

Determine the equation of the line that passes through points (2,4) and (6,1)

Answers

Answer:

y = -3/4x + 11/2

Step-by-step explanation:

y2 - y1 / x2 - x1

1 - 4 / 6 - 2

-3 / 4

-3/4

y = -3/4x + b

4 = -3/4(2) + b

4 = -3/2 + b

11/2 = b

Evaluating an Expression
Evaluate (15 - 4x) + 5 when x = 3.

Answers

Answer:

8

Step-by-step explanation:

(15-4x)+5

(15-4(3))+5

(15-12)+5

3+5=8

Answer:

seven

Step-by-step explanation:

-3/8p=9 how do you isolate p?

Answers

Answer:

multiply both sides by 8 then divide by -3

or in one step divide both sides by (-3/8) which is the same as multiplying by

(-8/3)...

-3p = 72

p = -24

Step-by-step explanation:

what’s the answer to this one also

Answers

Answer:

RS = 43

Step-by-step explanation:

Since RT is the whole line, your total is 59. Since you have part of what RT is which is 16, subtract 16 from 59.

59 - 16 = 43

Ayuda porfavor es un examen de ahorita :(

Answers

Answer:

hi

Step-by-step explanation:

#17: If the area of a rectangle measures 20 square inches, the side lengths must
be 4 inches and 5 inches.

If the statement above is false, provide a counterexample.

Answers

A counterexample could be 2 inches and 10 inches.

slove the following equation.
6n - 3(n + 2) + 3(7n + 1) = 0​

Answers

Answer:

here is my working out to find answer, if u feel that the answer is wrong then do the reverse calculation by putting n value to the equation. hope u understood and stay safe!

If a + b = 20 and ab = 75, find the value of a? + b2.​

Answers

Answer:

a² + b² = 250

Step-by-step explanation:

Given

a + b = 20 ( square both sides )

(a + b)² = 20² , that is

a² + b² + 2ab = 400 ( substitute ab = 75 )

a² + b² + 2(75) = 400

a² + b² + 150 = 400 ( subtract 150 from both sides )

a² + b² = 250

Select the correct answer for each statement.
1) will yield consecutive odd integers.
2k+9, 2k+11, 2k+13
or
3k+2, 3k+3 , 3k+4

2)will yield consecutive integers.
k+6, k+8 , k+10
or
k+1, k+2, k+3

Answers

Answer:

Part A. Option a. is correct

Part B. Option a. is correct.

Step-by-step explanation:

Part A.  

The numbers given by 2k + 9, 2k + 11, 2k + 13 will represent 11, 13 and 15 for k = 1, which are consecutive odd integers.

And the numbers given by 3k + 2, 3k + 3 and 3k + 4 will represent 5, 6, and 7 for k = 1 and they are not consecutive odd integers.

Therefore, option a is correct.

Part B.  

The numbers given by k + 1, k + 2, k + 3 will represent 2, 3 and 4 for k = 1, which are consecutive integers.

And the numbers given by k + 6, k + 8, and k + 10 will give 7, 9, and 11 for k = 1 and they are not consecutive integers.

Therefore, option A is correct.  

Step-by-step explanation:

Hope it helps

Answer:

1 is a. and 2 is a.

Step-by-step explanation:

look at the guy above me

-m/3=-4
m?
Yeah i need to figure out how to do this one

Answers

Answer:

assuming that this is just an algebra problem and not some graphing

problem that you are trying to fin the slope vale m in y-mx+b

-m/3 = -4

-m = -4*3

-m = -12

m=12

Step-by-step explanation:

70° ? 127° 146° Can I get some help with this?​

Answers

Answer:

93 degrees

Step-by-step explanation:

First, you would need to observe the image and see where to start off. You can see that there is a triangle below the "?" and you would be able to get the answer if you can find the measures of the small triangle(because of vertical angles) So,

180-127= 53.

180-146=34.

Since the angles of a triangle adds up to 180 degrees, 53+34+x=180.

x=93

Now, just flip the angle around and "?" = 93 degrees.

Answer:  93 degrees

===========================================================

Explanation:

Let x be the question mark angle we want to find.

Let y be adjacent to the 127 degree angle

Let z be adjacent to the 146 degree angle.

Refer to the diagram below.

Note that in the diagram, I have a second copy of 'x' because the two x angles are vertical angles. Vertical angles are always congruent.

Let's find angle y

127+y = 180

y = 180-127

y = 53

Do the same for angle z

z+146 = 180

z = 180-146

z = 34

Then notice how,

x+y+z = 180

x+53+34

x+87 = 180

x = 180-87

x = 93

Which is true when a negative number is subtracted from a positive number?
A. The difference is always zero.
B. The difference is always positive.
C. The difference is always negative,
D. The difference could be positive or negative

Answers

Answer:

D is the correct answer

Step-by-step explanation:

it honestly depends on what the numbers are

for example, 12 -6, you would get 6.

but if the question is something like 4 -8, you'd have -4

Subtracting a number is the same as adding its opposite. So, subtracting a positive number is like adding a negative; you move to the left on the number line. Subtracting a negative number is like adding a positive; you move to the right on the number line.
So it’s d

(4xz)³-2xy

x= 2 y=(-4) z= 1

plz help it's for homwork.​

Answers

Answer:

528

Step-by-step explanation:

Plug in values.

(4 x 2 x 1)³ - 2(2)(-4)

8³ - (-16)

512 + 16

528

Sandi got 1/1/4 pounds of gummy worms if she left the store with 3 pounds of gunmys worms and skittles ,how many pounds of skittles did she get

Answers

Answer:

1 3/4 lbs of skittles

Step-by-step explanation:

Easy way: 3lbs - 1 1/4lbs = 1 3/4lbs

Complex way:

(I'm assuming "1/1/4" means "1 1/4")

This is simply ~Subtraction~

The total is 3lbs(t) and 1 1/4 of that 3lbs is gummy worms(g). What is left of the 3lbs is skittles(s).

So we need to know what added to 1 1/4 equals 3.

Equation: t= 1 1/4 + s

Equation: t= 1 1/4 + s

We can replace (t) with 3 because we know this variable.

1. 3= 1 1/4 + s

To isolate the variable (s) we need to subtract 1 1/4 on both sides of the equation

2. 3(- 1 1/4)= 1 1/4(- 1 1/4) + s

The right side cancels out = 0 and the left = 1 3/4

3. 1 3/4= s

Flip it around for the traditional view

4. s = 1 3/4

In word form: Skittles equal 1 3/4 (don't forget to add lbs or pounds back into the final answer)

The amount Ricky charges for his lawn

Answers

Answer: 187.50 for 5 hours

112.50 ÷ 3 = 37.5037.5 × 5 = 187.50

I believe that's how they want it done.

Check Your Un
4. Suppose you are asked to solve the equation 3/4x - 2/3=1/6x
a. What number could you multiply both sides of the equation by so
that the numbers in the problem are integers and not fractions?
b. What property allows you to do this?

Answers

Answer:

L.C.M of 4,3 and 6 is 12

The Multiplication Property for Equations states that an equation can be multiplied or divided by the same number on each side of the equation without changing the solution to the equation.

Step-by-step explanation:

I need the awnser for 10 please help

Answers

[tex] \mathfrak{Solution : }[/tex]

The sum of two complementary angles is 90° , so

(5x + 7)° + (8x - 21)° = 90°

5x + 8x + 7° - 21° = 90°

13x - 14 = 90°

13x = 90° + 14°

13x = 104°

x = 104° ÷ 13

x = 8°

1st angle :

(5x + 7)°

((5×8) + 7)°

(40 + 7)°

47°

2nd angle :

(8x - 21)°

((8×8) - 21)°

(64 - 21)°

43°

A.53 degrees
B.143 degrees
C.37 degrees
D.90 degrees
???????????????????????????

Answers

Answer:

C. 37° Opposite angles

I hope I helped you^_^

PRECALCULUS WITH AB

At 0°C, the volume of a gas is 22 L. For each degree, the temperature T (in degrees Celsius) increases, the volume V (in liters) of the gas increases by 2/25

write an equation that represents the volume of the gas in terms of the temperature

Answers

the answer is 5/4 L temperate

(06.02 MC)
Which of the following correctly arranges the planets by the size of their diameter in increasing order?
1. Uranus < Neptune < Mercury < Earth
2. Earth < Mercury < Neptune < Uranus
3. Mercury < Earth < Saturn < Jupiter
4. Jupiter < Saturn < Earth < Mercury

Answers

Mercury←Earth←Saturn←Jupiter

Answer:

3-

Step-by-step explanation:

1. Find the measure of

Answers

Answer:

p = 70 degree

c = 70 degree

P degree
C degree
Please give me points

An angle measuring 60 degrees is measured as 62 degrees. What is the percentage error correct to 3 significant figures?

Answers

Answer:

3.33%

Step-by-step explanation:

60 = theoretical/actual degree measure

62 = experimental/calculated degree measure

% error = (|actual - calculated|/actual)*100

In words, the absolute value of the actual value minus the calculated value is first calculated. Then, divide the difference by the actual value that you should have obtained. Since you want this converted to a percent, multiply the decimal value of your quotient by 100.

Three significant figures is 3.33 because none of these numbers are leading or trailing zeros. If you count how many numbers (that are not leading or trailing zeros) the value 3.33 has, you'll find 3 significant figures, which is why your final answer should be 3.33%.